Difference between revisions of "2003 AMC 10B Problems/Problem 2"
m |
m (dollar sign) |
||
Line 1: | Line 1: | ||
==Problem== | ==Problem== | ||
− | Al gets the disease algebritis and must take one green pill and one pink pill each day for two weeks. A green pill costs <math> \ </math><math>1</math> more than a pink pill, and Al's pills cost a total of <math> \ | + | Al gets the disease algebritis and must take one green pill and one pink pill each day for two weeks. A green pill costs <math> \ </math><math>1</math> more than a pink pill, and Al's pills cost a total of <math>\textdollar 546</math> for the two weeks. How much does one green pill cost? |
− | <math> \textbf{(A) }\ | + | <math> \textbf{(A)}\ \textdollar 7 \qquad\textbf{(B) }\textdollar 14 \qquad\textbf{(C) }\textdollar 19\qquad\textbf{(D) }\textdollar 20\qquad\textbf{(E) }\textdollar 39 </math> |
==Solution== | ==Solution== | ||
Line 19: | Line 19: | ||
x&=20\end{align*}</cmath> | x&=20\end{align*}</cmath> | ||
− | Therefore, the cost of a green pill is <math>\boxed{\textbf{(D) } 20 | + | Therefore, the cost of a green pill is <math>\boxed{\textbf{(D) }\textdollar 20}</math>. |
==See Also== | ==See Also== | ||
{{AMC10 box|year=2003|ab=B|num-b=1|num-a=3}} | {{AMC10 box|year=2003|ab=B|num-b=1|num-a=3}} |
Revision as of 18:38, 26 November 2011
Problem
Al gets the disease algebritis and must take one green pill and one pink pill each day for two weeks. A green pill costs more than a pink pill, and Al's pills cost a total of for the two weeks. How much does one green pill cost?
Solution
Since there are days in weeks, Al has to take green pills and pink pills in the two week span.
Let the cost of a green pill be dollars. This makes the cost of a pink pill dollars.
Now we set up the equation and solve. Since there are pills of each color, the total cost of all pills, pink and green, is dollars. Setting this equal to and solving gives us
Therefore, the cost of a green pill is .
See Also
2003 AMC 10B (Problems • Answer Key • Resources) | ||
Preceded by Problem 1 |
Followed by Problem 3 | |
1 • 2 • 3 • 4 • 5 • 6 • 7 • 8 • 9 • 10 • 11 • 12 • 13 • 14 • 15 • 16 • 17 • 18 • 19 • 20 • 21 • 22 • 23 • 24 • 25 | ||
All AMC 10 Problems and Solutions |